6
$\begingroup$

Here is what looks like (but is not) an Olympiad problem. Is it really that tough, or am I overlooking a simple solution?

I have a system of sequences $\sigma_0(m),\sigma_1(m),\ldots\,$ defined for all integer $m\ge 0$ as follows. If $\min\{m,n\}=0$, then $\sigma_n(m)=0$. If both $m$ and $n$ are positive, then find the (uniquely defined) integers $K\ge 1$ and $j\in[0,n-1]$ such that $K^n\le m<(K+1)^n$ and indeed, $K^{n-j}(K+1)^j\le m<K^{n-j-1}(K+1)^{j+1}$, and writing $m=K^{n-j}(K+1)^j+R$, let $$ \sigma_n(m) := (Kn+n-j)K^{n-j-1}(K+1)^{j-1} + \sigma_{n-1}(R). $$ (The expression $(Kn+n-j)K^{n-j-1}(K+1)^{j-1}$ can be understood as the "formal derivative" $\frac{d}{dK}K^{n-j}(K+1)^j$.) Thus, for instance, $\sigma_1(m)=1$ for all $m\ge 1$, and $$ \sigma_2(m) = \begin{cases} 2K+1\ &\text{if}\ K^2<m\le K^2+K, \\ 2K+2\ &\text{if}\ K^2+K<m\le (K+1)^2. \end{cases} $$

Is it true that for all $k,n,m_1,\ldots,m_k\ge 1$, we have $$ \sigma_n(m_1+\dotsb+m_k) \le \sigma_{n-1}(m_1)+\dotsb+\sigma_{n-1}(m_k) + \max\{ m_1,\ldots,m_k \} ? $$

The cases where $n=2$ and $m_1=\dotsb=m_k=1$ follow without much effort, but already the general case where $k=1$ seems difficult:

Is it true that $\sigma_n(m)\le\sigma_{n-1}(m)+m$ for all integer $m,n\ge 1$?

Computations seem to confirm the inequality in question.

$\endgroup$

1 Answer 1

2
$\begingroup$

So far I was not able to prove the inequality $\sigma_n(m)\leq\sigma_{n-1}(m)+m$ and have not even convinced myself that it is true. Still, I have deduced a number of properties that one may find helpful. I state them as lemmas below with only brief proof ideas, but can provide complete proof(s) upon request.

Everywhere below I assume $m,n\geq 1$ be integer and $K,R,j$ be defined as in the question.

Lemma 1. $$\sigma_{n}(m) = \frac{Kn+n-j}{K(K+1)}(m-R) + \sigma_{n-1}(R).$$

Proof. Trivial.

Lemma 2. $$R < \frac{m}{K+1} < m^{(n-1)/n} \leq \frac{m}{K}.$$

Lemma 3. For any fixed $n\geq 0$, $\sigma_n(m)$ is a non-decreasing function of $m$.

Proof idea. Use Lemmas 1,2 and induction on $n$.

Lemma 4. $$\frac{mn}{K+1} \leq \sigma_n(m) \leq \frac{mn}{K-1},$$ where the upper bound holds for $K>1$. For $K=1$ (i.e., $m<2^n$), we have $$\sigma_n(m) \leq mn.$$

Proof idea. Use Lemmas 1,2,3 and induction on $n$.

Lemma 5. $$\sigma_n(m) \geq \frac{(K+2)n-j-1}{(K+1)^2}m.$$ Furthermore, for $1<K\leq n$, we have $$\sigma_n(m) \leq \frac{K}{K^2-1}mn - \frac{j+1}{(K+1)^2}m \leq \frac{K}{K^2-1}mn,$$ while for $K=1$ (i.e., $m<2^n$), we have $$\sigma_n(m) \leq mn-\frac{j+2}{4}m \qquad (m>1).$$

$\endgroup$
2
  • $\begingroup$ Thanks for sharing, I'll try to see whether this (particularly Lemma 4) can be useful. Concerning monotonicity - I also believe that $\sigma_n(m)$ is strongly increasing in $n$ for any fixed $m\ge 1$, but cannot prove this. $\endgroup$
    – Seva
    Mar 2, 2016 at 9:11
  • $\begingroup$ BTW, as your Lemma 4 shows, we have $\sigma_n(m)\approx nm^{1-1/n}$, and I think the key to understanding $\sigma_n$ is to somehow make this approximation precise. $\endgroup$
    – Seva
    Mar 2, 2016 at 11:29

Your Answer

By clicking “Post Your Answer”, you agree to our terms of service and acknowledge that you have read and understand our privacy policy and code of conduct.

Not the answer you're looking for? Browse other questions tagged or ask your own question.